Calendrier de l’Avent

1246789

Réponses

  • Il me semble que les questions posées dans ce fil se trouvent toutes solutionnées sur MSE, etanche dans ce fil https://les-mathematiques.net/vanilla/index.php?p=/discussion/comment/2397348/#Comment_2397348 a révélé son arme redoutable, le site . Par curiosité, je l'ai essayé et j'ai trouvé  la question de Calli solutionnée. (Par commodité, je ne donne pas les liens) 
    Je "demande" aux poseurs des questions pour les jours qui viennent de s'assurer que la question n'est pas posée sur MSE avec l'arme fourni par etanche 
    Le 😄 Farceur


  • Comme Julia Paule, j'avais mal interprété la question : je pensais que l'on calculait les restes de proche en proche ; donc, pour $2^{2^{2^2}}$, on aurait calculé in fine $2^1$ alors que c'est $2^{16}$ sans réduction.
  • Merci @gebrane. En effet, cela enlève de l'intérêt, il me semble bien avoir vu passer le dernier exercice.
    Il est préférable que les questions posées soient des exercices personnels (de son cru), même s'ils sont plus accessibles ou qu'ils comportent des erreurs (à éviter quand même).
  • Bibix
    Modifié (December 2022)
    J'ai cherché mon exo avec la recherche  https://approach0.xyz/search/?q=OR content:$\ \tan\left(\sum_{k\ =\ 0}^{\infty}\arctan\left(\frac{\left(-1\right)^k}{2\ k\ +\ 1}x\right)\right)$&p=3
    Il semble que le moteur de recherche est plutôt mauvais en réalité. Je trouve de bien meilleurs résultats sur cette somme due à Ramanujan (je l'ai appris avec le lien d'etanche) avec le lien d'origine d'etanche qui posait la question dérivée.

    Bref, s'il faut transformer sa question pour savoir si c'est effectivement répondu sur MSE, autant l'ignorer. Il y aura forcément des réponses qui traiteront de sujets similaires. Quoique pour mon exo du 18, je n'en suis pas sûr... . En tout cas, les exos que j'ai préparé sont des choses que je n'avais jamais vu ailleurs que dans mes notes, je peux te l'assurer.

    Edit :J'ai cherché par curiosité mon exo du 18, et il semblerait qu'il soit complètement inconnu de MSE. Je n'ai rien trouvé qui soit même proche de mon exo.
  • Calli
    Modifié (December 2022)
    Euh non @gebrane, je ne suis pas d'accord. Si des gens veulent tricher en regardant sur MSE c'est leur problème. Ça ne devrait pas nous empêcher de faire les maths qu'on veut !

    PS : Désolé si mon énoncé était un peu ambigu. Je l'ai édité pour clarifier.
  • a) Ne demande pas d'excuse, Calli, il était suffisamment clir et c'est moi qui l'ai mal interprété.

    b) Dans un sous-forum concurrent, un forumeur très actif (il se reconnaîtra) a publié un exercice d'Algèbre qu'il comptait poser prochainement dans ce fil (mais une de mes interventions lui a savonné l'herbe sous le pied). Alors, je pose la question : dans ce fil, quelles catégories ont-elles droit de cité ? J'avais cru lire arithmétique, séries, intégrales,... mais non pas algèbre, proba, géométrie.
  • gebrane
    Modifié (December 2022)
    @Bibix voici l'ingrédient de ton exo https://math.stackexchange.com/questions/1579867/how-to-find-sum-n-0-infty-sum-k-0-infty-frac-1nk2k12n12?noredirect=1
    Il suffit d'entrer les bons mots clefs dans le moteur . J'ai demandé seulement avant de poster son exo de vérifier s' il était traité dans MSE  (Egalité-fraternité) :D  

    Ajout Oui @john_john j'étais choqué de voir mon exo écrasé par ta question initiale sur ledit fil  ( même si la forme des deux questions est différente ) 
    Le 😄 Farceur


  • Snif, snif, gebrane !  En fait, mon exo ne grillait pas le tien car, si MrJ n'avait pas donné de solution, j'aurais proposé la mienne et elle était sans rapport avec ton exo.
  • JLapin
    Modifié (December 2022)
    Le cas $a=1$ est évident. Supposons $a\geq 2$.
    Lemme
    Soit $m$ et $q$ deux entiers naturels non nuls. Alors, la suite $(m^k \mod q)_{k\geq 1}$ est périodique à partir d'un certain rang, de périodique inférieure ou égale à $q-1$.
    En effet, si $m$ et $q$ sont premiers entre eux, alors $[m]_q$ appartient au groupe des inversibles de l'anneau $\Z/q\Z$, donc $(m^k \mod q)_{k\geq 1}$ est périodique de période l'ordre de $[m]_q$ dans ce groupe qui est de cardinal strictement plus petit que $q$.
    Si $m$ et $q$ ne sont pas premiers entre eux, alors la suite finie $(m^k \mod q)_{k\in \llbracket 1,q\rrbracket}$ est à valeurs dans $\llbracket 0,q-1\rrbracket \setminus \{1\}$ (sinon, Bezout...). Il existe donc $i<j$ dans $\llbracket 1,q\rrbracket$ tel que $m^i=m^j[q]$ et donc, $(m^k)_{k\geq i}$ est périodique de période au plus $j-i\leq q-1$.
    Le résultat
    On note $p_1$ la période (à partir d'un certain rang) de la suite $(a^k \mod n)_{k\geq 1}$ avec $p_1<n$.
    Si $p_1=1$, il n'y a plus rien à faire.
    Si $p_1\geq 2$, on note $p_2<p_1$ la période (à partir d'un certain rang) de la suite $(a^k \mod p_1)_{k\geq 1}$.
    Si $p_2=1$, pour $a$ assez grand la tour $a^a...$ possède un résidu constant modulo $p_1$ et donc la tour initiale $a^a...$ possède un résidu constant modulo $n$.
    Si $p_2\geq 2$, on recommence.
    Comme il n'existe pas de suite strictement décroissante d'entiers naturels, au bout d'un nombre fini d'étapes, on tombe sur une période qui vaut $1$, ce qui montre par "descente successives" que la suite des résidus des $u_k$ modulo $n$ stationne à partir d'un certain rang (cette partie de la preuve ne sera pas rédigée).
    Jour 6.
  • etanche
    Modifié (December 2022)
    @ gebrane si on veut avoir des exercices inédits le mieux c’est de les inventer. 
    Pour des exercices complètement nouveaux, Il faudrait un comité de sélection expert pour vérifier que l’exercice n’existe pas dans des livres, revues, maths stackexchange, mathsOverflow, approach0xyz. 
    Est-ce le but du calendrier proposer des inédits ? Est-ce une compétition (celui qui résout le plus rapidement) ? 
    Des exercices que seuls 3 ou 4 personnes savent résoudre ? 
  • Le but est simple : des exercices inédits, des exercices avec un énoncé plutôt simple(potentiellement résoluble par tout le monde, même si des outils avancés permettent la résolution rapide) Avec(si possible) une jolie solution :) 
    Bien sûr on joue le jeu on ne regarde par la solution sur internet ou des trucs comme ça, aussi les solutions donnés en 30 secondes c'est plutôt triste... Donc les solutions devront êtres posté a partir de 16h, et les avancées non réussie(ou des idées d'angle d'attaque), quand on veut 
    Je suis donc je pense 
  • JLapin
    Modifié (December 2022)

    Jour 7

    Calculer $\displaystyle \sum_{n=1}^{+\infty} \frac{1}{n} \int_{2n\pi}^{+\infty} \frac{\sin t}{t} dt$.
  • Et oui, cette intégrale est référencée mais évidemment, personne n'est obligé d'aller voir la réponse avant de faire le calcul !
    https://approach0.xyz/search/?q=OR content:$\sum_{n=1}^{\infty}\frac{1}{n}\cdot\int_{2\cdot n\cdot pi}^{+\inf ty}\frac{\sin\left(x\right)}{x}dx$&amp;p=1
  • Magnéthorax
    Modifié (December 2022)
    @john_john : j'ai reçu une autorisation pour une question de probas élémentaires (pile ou face).

    @Quentino37 : vendredi 9, je posterai entre 8h30 et 9h30, pas à 8h, car le matin est chargé : domicile + école + collège en 1h15.
  • Voici quelques avancées (le texte est encore rudimentaire et en cours de maturation).
    Plus j'avance et plus j'ai envie d'ajouter des éléments. On verra si j'arrive à faire des choix.

    Les commentaires sont les bienvenus.
  • Bonjour @rémi  peux-tu enlever les réponses fausses. Exemple la mienne
    Le 😄 Farceur


  • Oui @gebrane, ce n'est pas encore fait [c'est l'ours si on peut dire]. Je ferai une épuration au fur et à mesure.
  • gebrane
    Modifié (December 2022)
    J-lapin puisque c'est fait sur MSE, je passe mon tour  :(
    Le 😄 Farceur


  • Au contraire gebrane il faut trouver une solution différente de celle existante à ce jour. 
  • Justement ce que je voulais proposer est déjà fait sur MSE. 
    Le 😄 Farceur


  • Julia Paule
    Modifié (December 2022)
    @JLapin pour le J6
    Je ne comprends pas pourquoi tu parles de la suite $(a^k)$ et non pas de la suite $(u_k)$, tour des $a$ dont il est question dans l'énoncé. Par exemple tu dis que si $p_1=1$, alors il n'y a plus rien à faire. Ceci veut dire qu'à partir d'un certain rang, on a $a^{k+1}=a^k$ (et non pas $u_{k+1}=u_k$), cela ne donne rien.
  • Je m’étonne de ces réflexes d’aller voir ailleurs tout de suite. Ce n’est pas une leçon de morale ! Je parle du plaisir de chercher. Certains ne semblent pas l’apprécier autant que moi. Peut-être que je mets le plaisir de chercher au dessus du plaisir de trouver. Bien entendu, l’apothéose est la conjonction des deux. 
  • @rémi Pour ton document, la solution du jour 4 a été donnée par @gai requin. :)
  • JLapin
    Modifié (December 2022)
    Si la suite $(a^k \mod n)_{k\geq k_0}$ est constante, a fortiori n'importe quelle tour assez grande de puissance de $a$ sera congrue à $a^{k_0}$ modulo $n$ vu que l'exposant est lui-même une tour de puissances de $a$ donc va toujours dépasser le seuil $k_0$ à partir d'une certaine hauteur.
  • Julia Paule
    Modifié (December 2022)
    Pour la suite $(a^k \pmod n)$, c'est certain. Si $u_{k+1} \equiv u_k \pmod n$, alors $a^{u_{k+1}} \equiv a^{u_k} \pmod n$, aussi.
    Elle est périodique, de période au plus $n$ (je ne vois pas l'intérêt de démontrer qu'elle l'est de période au plus $n-1$).
    Ensuite il me semble que la période des $a^k \pmod n$ n'a rien à voir avec celle de la tour des $a$, en particulier, je ne vois pas pourquoi elle la diviserait. Par exemple, on peut avoir $a^5=a \pmod n$ donc période $4$, mais si $a=7$, cela ne donne rien pour la tour.
    EDIT : je crois comprendre, tu ne t'intéresses pas à la tour dans un 1er temps, mais seulement aux puissances de $a \pmod n$, et tu dis qu'à partir d'un certain rang, la suite des puissances est constante ? Alors on a $p_1 \mid n$, sinon je ne vois pas le lien entre les modulos $n$ et les modulos $p_i$ ?
  • JLapin
    Modifié (December 2022)
    Mon explication est apparemment mal passée.
    Prenons l'exemple d'une tour de $7$ que l'on réduit modulo $10$.
    La suite $(7^k \mod 10)_{k\geq 1}$ est périodique de période 4.
    Ensuite, la suite $(7^j \mod 4)$ est périodique de période $2$.
    Enfin, la suite $(7^i \mod 2)$ est constante (périodique de période 1).
    Ainsi, pour n'importe quelle tour de $7$ que l'on note $u_k$, on a $u_k \equiv 1[2]$.
    Puis, $u_{k+1}=7^{u_k} \equiv 7[4] \equiv 3[4]$ (par $2$-périodicité).
    Enfin, $u_{k+2} =7^{u_{k+1}}\equiv 7^3 [10] \equiv 3[10]$ (par $4$-périodicité).
    Donc la suite des tours de $7$ réduite modulo 10 stationne à $3$ à partir du rang $3$.
    Exercice (fabriqué au hasard, normalement il n'y a pas la réponse dans le site préféré de certains).
    Calculer la réduite modulo $50$ de $13^{13^{13}}$ (calculatrice à 4 opérations autorisée).
    Trouver un rang à partir duquel on est sûr que la suite des tours de $13$ stationne modulo 50.
  • gebrane
    Modifié (December 2022)
    Dom je crois que tu me vises dans ceci "Je m’étonne de ces réflexes d’aller voir ailleurs tout de suite." J'ai dit, j'ai cherché  et trouvé mais Jlapin a donné  un lien vers des  solutions. J'ai préféré  ne rien poster  puisque  c'est fait dans un lien.
    Le 😄 Farceur


  • Quentino37
    Modifié (December 2022)
    Vu que étanche n’a pas le temps de poster son exo demain, il m’a demandé de poster son exo à sa place (je le fais un peu en avance mais ce n’est pas grave… on attend 16h pour les réponses complètes).

    Jour 8


    Pour toute fonction $f$ de $\mathbb R$ dans $\mathbb R$ montrer qu’il existe un réel $a$ tel que le graphe de $h = f-a$ ne contient aucun point à coordonnées rationnelles.
    Je suis donc je pense 
  • Et il en existe même beaucoup :)

    Bel exo ! Je note...
  • Julia Paule
    Modifié (December 2022)
    @JLapin Ah d'accord, les modulos portent sur les exposants, je n'avais pas compris car dans le cas particulier d'une tour, les exposants sont tous égaux au nombre de départ $a$. J'avais déjà remarqué, sans approfondir, que dans les exercices portant sur "réduire $x^y$ modulo $n$" apparait un nombre, que tu appelles donc la période de $x$ modulo $n$, qui permet de résoudre ce genre d'exercices. On peut donc généraliser pour réduire $x^{{y^z}^{\cdots}}$ modulo $n$, en regardant la période $p_1$ de $x$ modulo $n$, puis la période $p_2$ de $y$ modulo $p_1$, puis la période $p_3$ de $z$ modulo $p_2$, etc ...
    Cela répond à une question que je me posais depuis longtemps.
    Dans ce cas général, il n'y a aucune raison que les périodes soient strictement décroissantes. C'est pour ça que tu montres que $p_1 < n$. Par récurrence, dans le cas d'une tour, on aura une période qui va arriver à $1$.
    Dans le cas général, il y a une puissance incompressible, c'est la 1ère puissance $k_1$ où apparait la période $p_1$, i.e. pour $y-k_1=\lambda p_1+r_1$, avec $0 \leq r_1 < p_1$, on aura $x^y=x^{k_1+\lambda p_1+r}=x^{k_1+r_1} \pmod n$. Je ne suis pas sûre de savoir ce qu'elle devient.
    Dans le cas de la tour, il y a en a une également. Dans ton exemple avec $7 \wedge 10 =1$, il n'y en a pas car $7$ est inversible modulo $10$.
    Par exemple avec $14^{{14^{14}}^{\cdots}}$ modulo $24$, la période est $2$ et la 1ère puissance incompressible est $3$. On a : $14^2=4, 14^3=8, 14^4=16, 14^5=8$.
    Bon, je vais faire ton exercice et approfondir les cas, j'en vois 3 :
    $a \wedge n=1$ (ok pour ça car il n'y a pas de puissance incompressible),
    $a \wedge n \ne 1$ tous deux divisés par les mêmes nombres premiers (ok aussi car on aura une puissance nulle à partir d'un certain rang, donc périodique),
    $a \wedge n \ne 1$ et pas divisés par les mêmes nombres premiers : à voir.
    Je déborde dans ce fil, si vous estimez préférable que j'en ouvre un autre pour ne pas encombrer, faites-moi signe !
  • gebrane
    Modifié (December 2022)
    Je serais occupé, j'envoie de suite il se fait en une ligne sauf étourderie. Soit gebrane est bébête, soit  l'exo est bébête.    :D
    On cherche $a$ tel que $\forall r,s\in\mathbb Q,\  f(r)-a\neq s$, ce qui est équivalent à $\forall r,s\in\mathbb Q, \ a\neq f(r)-s$, on choisit $a$ dans le complémentaire de l'ensemble $\{ f(r)-s\mid r,s\in\mathbb Q\}$.
    Le 😄 Farceur


  • En gros, c'est un peu style Ulm... Un énoncé simple, à priori, mais qui demande réflexion... 
  • Ni gebrane, ni l'exo ne sont bébêtes 
  • JLapin
    Modifié (December 2022)
    Voici ma preuve pour le jour 7.
    Par le changement de variable $t=nx$ puis par intégration par parties, on a $u_n=\frac{1}{n}\int_{2n\pi}^{+\infty} \frac{\sin t}{t} dt = \frac{1}{n} \int_{2\pi}^{+\infty} \frac{\sin(nx)}{x}dx = \int_{2\pi}^{+\infty} \frac{1-\cos(nx)}{n^2 x^2}dx$, ce qui montre déjà que la série $\sum u_n$ est convergente puisque $u_n=O(1/n^2)$.

    On pose $f_n(x)=\frac{1-\cos(nx)}{n^2 x^2}$ et on a $\int_{2\pi}^{+\infty} |f_n(x)|dx\leq \frac{2}{n^2}\int_{2\pi}^{+\infty} \frac{dx}{x^2}$, ce qui justifie l'intégration terme à terme et donc $\sum_{n=1}^{+\infty} u_n = \int_{2\pi}^{+\infty} f(x)dx$ avec $f(x)=\frac{1}{x^2}\sum_{n=1}^{+\infty} \frac{1-\cos(nx)}{n^2}$.

    On pose $g(x)=\sum_{n=1}^{+\infty} \frac{1-\cos(nx)}{n^2}$ (fonction $2\pi$-périodique) et, par la relation de Chasles et périodicité, on a $\int_{2\pi}^{+\infty} f(x)dx = \sum_{n=1}^{+\infty} \int_{0}^{2\pi} \frac{g(s)}{(s+2n\pi)^2}ds$.

    Par un calcul de série de Fourier classique, on a $g(s)=\frac{1}{4}s(2\pi-s)$ pour tout $s\in [0,2\pi]$ et donc, $\int_{2\pi}^{+\infty} f(x)dx = \sum_{n=1}^{+\infty} \frac{1}{4}\int_0^{2\pi} \frac{s(2\pi-s)}{(s+2n\pi)^2}ds$.

    Un calcul de primitive de fractions rationnelles donne $\frac{1}{4}\int_0^{2\pi} \frac{s(2\pi-s)}{(s+2n\pi)^2}ds = \frac{\pi}{2}(x_{n}-x_{n-1})$ avec $x_n = (2n+1)\ln(n+1)-2\ln(n!)-2n$.

    Par Stirling, la suite $(x_n)$ converge vers $2-\ln(2\pi)$ et donc la somme télescopique de terme général $(x_n-x_{n-1})$ vaut $2-\ln(2\pi)-x_0=2-\ln(2\pi)$.

    En conclusion, on a $\sum_{n=1}^{+\infty} \frac{1}{n} \int_{2n\pi}^{+\infty} \frac{\sin t}{t}dt = \frac{\pi}{2}(2-\ln(2\pi)) = \pi - \frac{\pi}{2}\ln(2\pi)$.
  • JLapin
    Modifié (December 2022)
    Jour 8 (désolé pour le délai non respecté).
    Soit $f$ une fonction de $\R$ dans $\R$.
    Notons $A$ l'ensemble des $a\in \R$ tels qu'il existe $x\in \Q$ vérifiant $f(x)-a\in \Q$. On a alors $A=\displaystyle \bigcup_{(x,r)\in \Q^2}\{f(x)-r\}$.
    En tant que réunion dénombrable de singletons, $A$ est une partie dénombrable de $\R$ donc une partie stricte de $\R$.
    On choisit $a\in \R\setminus A$ et le graphe de $f-a$ ne contient aucun point à coordonnées rationnelles.
  • Oh flûte ! Ca fait deux fois que je suis drôlement intéressé par un exo pour mes élèves... et finalement, je ne peux pas le donner à cause de la présence de séries de Fourier à un moment crucial.
  • etanche
    Modifié (December 2022)
    Pour J8 j’ai proposé pas difficile et faisable pour les élèves de Sup (voir aux terminales en faisant un problème avec questions intermédiaires sur la dénombrabilité).
  • JLapin
    Modifié (December 2022)

    Tu peux Je pensais que l'on pouvait remplacer le calcul par série de Fourier par l'étude de la série entière $\sum_{n=1}^{+\infty} \frac{1-\cos(ns)}{n^2} t^n$ pour $s\in ]0,2\pi[$ fixé mais en fait, les calculs semblent trop pénibles...
    La somme est continue sur $[0,1]$ par convergence normale et la dérivation permet de la calculer effectivement sur $[0,1[$

    Je propose ceci :

    On pose $g_n(x)=\frac{1-\cos(nx)}{n^2}$ et $g(x)=\sum_{n=1}^{+\infty} g_n(x)$.
    On vérifie que la série $\sum g_n'$ converge uniformément sur tout segment de $]0,2\pi[$ (transformation d'Abel), ce qui permet de dériver terme à terme sur $]0,2\pi[$.
    On trouver $g'(x)=\sum_{n=1}^{+\infty} \frac{\sin(nx)}{n}$ et cette fois, les calculs à partir de la série entière $S(t)=\sum_{n=1}^{+\infty} \frac{\sin(nx)}{n} t^n$ sont faisables sur $]0,1[$, puis par continuité de cette série entière en $1$ (théorème d'Abel radial), on obtient $g'(x)=S(1)=\frac{\pi}{2} -\frac{x}{2}$.

    Il n'y a plus qu'à intégrer pour récupérer $g(x) = \frac{1}{4}x(2\pi-x)$.

  • Bibix
    Modifié (January 2023)
    Les séries de Fourier, c'est pour ne pas se casser la tête... . Pour le jour 3 par exemple, on peut considérer : 
    $f_x(t) = \sum_{k = 0}^{+\infty} \frac{(-1)^k (2k+1)}{x^2 + (2k+1)^2} e^{-(2 k + 1) t}.$
    On obtient alors l'équation différentielle après de pompeuses justifications pour pouvoir dériver 
    $f_x''(t) = \sum_{k=0}^{+\infty} (-1)^k (2k+1) e^{-(2 k + 1) t}  - x^2 f_x(t) = g(t) - x^2 f_x(t).$
    Avec $- \int_u^{+\infty} g(t) dt = \frac{e^{-u}}{1 + e^{-2u}}$ pour tout $u > 0$ (oui, c'est assez pénible). Et donc $f_x$ vérifie 
    $\forall t > 0, \quad f_x''(t) + x^2 f_x(t) = \frac{d}{dt}\left(\frac{e^{-t}}{1 + e^{- 2 t}}\right)(t).$
    Normalement, on retombe sur le même résultat.

    Edit (Janvier) : Finalement, ce yakafokon ne convient pas. Voici une solution sans analyse complexe (quasiment). On commence par calculer pour $n \in \mathbb{N}^*$ et $t \in ]0, \pi[$, $$\sum_{k = 1}^{n-1} (-1)^{k-1}\frac{\sin(k t)}{k} = \int_0^t 1 - \sum_{k = 0}^{n-1} (-1)^{k} \cos(k x) dx = \frac{t}{2} + \frac{(-1)^n}{2} \left(\frac{\sin(n t)}{n} + \int_0^t \frac{\sin(n x) \sin(x)}{1 + \cos(x)}dx\right).$$ Avec une IPP, on en déduit $$\sum_{k = 1}^{n-1} (-1)^{k-1} \frac{\cos(k t)-1}{k^2} = \frac{t^2}{4} + \frac{(-1)^n}{2} \left(- \frac{\cos(n t)}{n^2} + \int_0^t (t-x) \frac{\sin(n x) \sin(x)}{1 + \cos(x)} dx\right),$$ puis $$\sum_{k = 1}^{n-1} (-1)^{k-1} \frac{\sin(k t)}{k^3} = t S_n -\frac{t^3}{12} - \frac{(-1)^n}{2} \left(- \frac{\sin(n t)}{n^3} - \int_0^t (t-x)^2 \frac{\sin(n x) \sin(x)}{1 + \cos(x)} dx\right),$$ où $S_n = \sum_{k = 1}^{n-1} \frac{(-1)^{k-1}}{k^2} \underset{n \to +\infty}{\to} \ell \in \mathbb{R}$. Avec une dernière IPP, on montre $$\sum_{k = 1}^{+\infty} (-1)^{k-1} \frac{\sin(k t)}{k^3} = \ell t - \frac{t^3}{12}.$$ Soit $x \in \mathbb{R}$, on pose $$f_x : t \in \mathbb{R} \longmapsto \sum_{k = 1}^{+\infty} \frac{(-1)^{k-1}}{x^2 + k^2} \frac{\sin(k t)}{k},$$ en remarquant que pour tout $t \in ]0, \pi[$, $$f_x(t) = \sum_{k = 1}^{+\infty} (-1)^{k-1} \left(1 - \frac{x^2}{x^2 + k^2}\right) \frac{\sin(k t)}{k^3} = \ell t -\frac{t^3}{12} - x^2 \sum_{k = 1}^{+\infty} \frac{(-1)^{k-1}}{x^2 + k^2} \frac{\sin(k t)}{k^3}.$$
    En dérivant deux fois, on obtient $f_x''(t) = -\frac{t}{2} + x^2 f_x(t)$, et avec les passages à la limite, on obtient $\lim_{t \to 0^+} f_x(t) = \lim_{t \to \pi^-} f_x(t) = 0$. Tout ceci se justifiant assez simplement par convergence normale des séries avec les théorèmes qui vont bien. On obtient alors $$\forall t \in ]0, \pi[, \quad f_x(t) = \frac{1}{x^2} \left(\frac{t}{2} - \frac{\pi \sinh(x t)}{2 \sinh(x \pi)}\right).$$ Il suffit ensuite de remonter vers ce qui nous intéresse en remarquant que $$\frac{\pi}{4}-x^2 f_x\left(\frac{\pi}{2}\right) = \sum_{k = 0}^{+\infty} \frac{(-1)^k}{2 k + 1} - x^2 \sum_{k = 0}^{+\infty} \frac{(-1)^k}{((2k+1)^2+x^2)(2k+1)} = \sum_{k = 0}^{+\infty} \frac{(-1)^k (2k+1)}{(2k+1)^2 + x^2}.$$
  • gebrane
    Modifié (December 2022)
    Par curiosité j'ai testé le moteur diabolique de étanche pour savoir si son exercice est original, Hélas , c'est  déjà traité https://artofproblemsolving.com/community/c7h1586290p9813654
    J'ai hésité de poser une  question à JLapin, est ce que tu as copié ton exo de J7 de ma question sur ce fil ( il y a une petite différence) https://les-mathematiques.net/vanilla/index.php?p=/discussion/comment/2393389/#Comment_2393389
    Le 😄 Farceur


  • @ gebrane  l’exos que j’ai proposé n’était pas inédit. 
  • gebrane
    Modifié (December 2022)
    Pour @bisam Tu peux proposer à tes étudiants l'exo du jour7  sans les séries de Fourier 
    Exercice 
    Partie 1 on démontre que $\sum\limits_{k=1}^{+\infty}\frac{\sin kx}{k}=\frac{\pi-x}{2}\qquad \forall x \in ]0,2\pi[$.
    Soit $x \in ]0,2\pi[$, 
    a) Montrer que $\sum\limits_{k=1}^{n}\cos kt=\frac {\sin (nx +x/2)}{2\sin (x/2)}-\frac {1}{2}.$
    b) En utilisant que $\int_0^x \cos(kt) dt=\frac{\sin kx}{k}$, montrer que $\sum\limits_{k=1}^{n}\frac{\sin kx}{k}=-\frac{x}{2}+I_n(x)+J_n(x)$ avec 
    $I_n(x)=\int_{0}^{x}\left(\frac{1}{2\sin{\frac{t}{2}}}-\frac{1}{t}\right)\sin \frac{(2n+1)t}{2}\ dt $ et $J_n(x)=\int_{0}^{x}\frac{\sin \frac{(2n+1)t}{2}}{t}dt$
    c) Montrer que en utilisant  $\int_{0}^{\infty}\frac{\sin s}{s}\ ds=\frac{\pi}{2}$ que  $\lim_{n\to +\infty} J_n(x)=\frac {\pi}2$
    d) Montrer que $\lim_{n\to +\infty} I_n(x)=0$ et en déduire que $\sum\limits_{k=1}^{+\infty}\frac{\sin kx}{k}=\frac{\pi-x}{2}$
    Partie 2 Dans cette partie on calcule $\displaystyle \sum_{n=1}^{+\infty} \frac{1}{n} \int_{2n\pi}^{+\infty} \frac{\sin t}{t} dt$
    Soit $S = \sum_{n=1}^\infty\frac{1}{n}\int_{2n\pi}^\infty\frac{\sin x}{x}dx$
    a) Montrer que $S= \sum_{n=1}^\infty \int_{2\pi}^{\infty} \frac{\sin nt}{nt}dt$
    b) En déduire que $S =\lim_{k\to\infty} \left(\sum_{n=1}^k \frac{1}{n}\int_0^\infty \frac{\sin nt}{t}~dt\right) -\lim_{k\to\infty} \left(\sum_{n=1}^{\infty} \int_{\frac{1}{k}}^{2\pi} \frac{\sin nt}{nt}~dt\right)$
    c) Montrer que $\sum_{n=1}^{\infty} \int_{\frac{1}{k}}^{2\pi}\frac{1}{t}\cdot \frac{\sin nt}{n}~dt= \frac{\pi}{2}\left(\log(2\pi)+\log(k)\right)- \pi + \frac{1}{2k}$
    d) Montrer que $\left(\sum_{n=1}^k \frac{1}{n}\int_0^\infty \frac{\sin nt}{t}~dt\right)= \frac{\pi}{2}\cdot H_k $
    e) En déduite la valeur de $S$.
    Le 😄 Farceur


  • etanche
    Modifié (December 2022)
    Dans le problème ci-dessus gebrane admet $\int_{0}^{+\infty} \frac{\sin(t)}{t} dt =\frac{\pi}{2}$ 
    voici une preuve en 4 questions http://www.math.ualberta.ca/~thillen/math300/integral.pdf de l’intégrale de Dirichlet.
  • Bien sûr etanche, il peut rallonger l'exercice en détaillant plus 
    Le 😄 Farceur


  • @JLapin
    Ton exercice du jour 7 a déjà été proposé sur le site en février 2019 : Série-intégrale de Dirichlet
  • Julia Paule
    Modifié (December 2022)
    ...
  • JLapin
    Modifié (December 2022)
    @jandri
    Effectivement : je vois que je n'ai pas trouvé une preuve très novatrice ! :smile:
    @Julia Paule"
    De rien !
  • Magnéthorax
    Modifié (December 2022)

    Jour 9

    Confinement dans les entiers naturels

    Soit $p\in\left]0,1\right[$. On note $q=1-p$. On suppose $p>q$. Soit $\left(X_n\right)$ une suite de variables aléatoires indépendantes et identiquement distribuées telle que $\mathbf{P}\left(X_n=1\right)=p$ et $\mathbf{P}\left(X_n=-1\right)=q$ pour tout $n\in\mathbf{N}^*$. On introduit $\left(S_n\right)$, la suite de variables aléatoires définie par : pour tout $n\in\mathbf{N}^*$, $S_n=\sum_{k=1}^n X_k$.
    Exprimer la probabilité $\mathbf{P}\left(\forall n\in\mathbf{N}^*,\ S_n\geq 0\right)$ à l'aide de $p$ et $q$.
  • Bonjour, pour le J6, complétement idiot mon truc plus haut avec les 3 cas, surtout qu'on peut avoir $a \wedge n =1$ mais $a \wedge p_k \ne 1$ (enfin, rien ne l'empêche a priori), il faut donc avoir un raisonnement plus général.
    En fait, non seulement la période pour $a$ modulo $n$ est $\leq n-1$, mais aussi la puissance incompressible $p$ : $\exists p<q \leq n-1$ tels que $a^p=a^q$, et idem pour les étages supérieurs modulo $p_k$. Donc dès qu'on atteint la période $1$, toutes les puissances supérieures sont égales à $u_k^p$, et c'est fini, d'où la périodicité, et c'est encore plus simple.
    C'est ce que tu dis, @JLapin dans ta preuve. Merci pour tes éclaircissements.
Connectez-vous ou Inscrivez-vous pour répondre.